Hi i need help with unit rate fractions and ill show an example i need to have this figured out by thurday for a test and i have no clue so pls help

Hi I Need Help With Unit Rate Fractions And Ill Show An Example I Need To Have This Figured Out By Thurday

Answers

Answer 1

To find the rate in teaspoons per cup we divide the number of teaspoons by the cups:

[tex]\frac{4}{\frac{2}{3}}=\frac{12}{2}=6[/tex]

Therefore, the unit rate is 6 teaspoons per cup.


Related Questions

Choose the point-slope form of the equation below that represents the line that passes through the points (-6, 4) and (2.0), (2 points)Oy - 4 = 2(x + 6)O y + 6 = 2(x - 4)Oy+6=-= (x-4)Oy-4--3(x+6)

Answers

Given the following:

(-6, 4) and (2, 0) are the lines that passes through the point.

we are asked to choose the point slope equation.

before we can solve it, we must first of all solve for the slope.

Slope m = y2 - y1

x2 - x1

where:

x1 = -6

x2 = 2

y1 = 4

y2 = 0

m = 0 - 4

2 - (-6)

m = -4/8

m = -1/2

The equation of the lines is found using the point-slope form:

y - y1 = m(x - x1)

so lets substitute into the above equation:

recall, y1 = 4, x1 = -6, slope m = -1/2

y - 4 = -1/2(x - (-6))

y - 4 = -1/2(x + 6)

Therefore, the equation of the lines using the point-slope form is:

y - 4 = -1/2(x + 6)

so the correct option is D which is y - 4 = -1/2(x + 6)

Solve: 6 · x=42What dose x=?

Answers

We have to solve this expression.

We can solve it dividing both sides by 6:

[tex]\begin{gathered} 6x=42 \\ \frac{6x}{6}=\frac{42}{6} \\ x=7 \end{gathered}[/tex]

Answer: x = 7

One custodian cleans a suite of offices in 8 hours. When a second worker is asked to join the regular custodian, the job takes only 4 hours. How long does it take thesecond worker to do the same job alone?The second worker can do the same job alone in___hours.

Answers

Given:

One custodian cleans a suite of offices in 8 hours.

So, the rate of the custodian to clean the office = 1/8

When a second worker is asked to join the regular custodian, the job takes only 4 hours.

So, the rate of both custodians = 1/4

Let the rate of the second custodian = x

So,

[tex]\frac{1}{8}+x=\frac{1}{4}[/tex]

Solve for x:

[tex]x=\frac{1}{4}-\frac{1}{8}=\frac{1}{8}[/tex]

So, the rate of the second custodian = 1/8

This means he will take 8 hours to clean the office alone

So, the answer will be:

The second worker can do the same job alone in 8 hours.

At the Dollar Spot, Carl bought pencils for $3.75, sharpies for $5 69, and glue sticks for ? 1. In the box below type which operation you would use: Division Addition Subtraction Multiplication 2. Why did you pick this operation?

Answers

Given that Carl bought pencils for $3.75, sharpies for $5 69, and glue sticks for

Although the question didn't give the value for glue sticks, the operation you would use here is Addition.

Addition symbol: +

2. I picked addition because to find the total amount Carl spent at the Dollar spot, you will need to add the amount he spent on pencils, sharpies and glue together.


995
× 55 ?? What’s the partial product of this?

Answers

The partial product is 52,525

For a certain kind of plaster work, 1.5 cu yd of sand are needed for every 100 sq yd of surface. How much sand will be needed for 350 sq yd of surface?

Answers

We are told that we need 1.5 cu yd of sand for every 100 sq yd of surface, then we can express the ratio of sand to surface like this:

[tex]\text{ratio}=\frac{1.5}{100}[/tex]

In order to find how much sand we need for 350 sq yd of surface, we just have to multiply 350 by this ratio, then we get:

[tex]350\times\frac{1.5}{100}=5.25[/tex]

Then, we need 5.25 cubic yards of sand.

Solve the following equation for "b".b/3 = M

Answers

In order to solve an equation for a variable we need to isolate it on the left side. In this case we want to find the value of "b", therefore we must perform operations in such a way that it will be the only thing on the left side of the equation. To do so we need to switch the operation of each term we don't want to be on the left side, this means that if a term is adding it should go to the right side subtracting and if it is multiplying it should go dividing. In this case there is only one term that is dividing "b", so it should go to the right side by multiplying. With this in mind lets solve the problem:

[tex]\begin{gathered} \frac{b}{3}\text{ = M} \\ b\text{ = 3}\cdot M \end{gathered}[/tex]

Lin is solving the inequality 15-x< 14. She knows the solution to the equation 15 - x = 14 is x = 1 How can Lin determine whether x > Torx < 1 is the solution to the inequality?

Answers

The inequality we have is:

[tex]15-x<14[/tex]

To solve this problem the x has to be positive, for this reason, the first step is to add x to both sides of the inequality:

[tex]15-x+x<14+x[/tex]

As you can see, on the left side -x+x is equal to 0. So we have:

[tex]15<14+x[/tex]

The next step to solving for x, now that the x is positive, is to leave the term with x alone on one side of the inequality. For this reason, we need to subtract 14 to both sides of the inequality:

[tex]15-14<14-14+x[/tex]

On the right side, 14-14 is equal to 0, and thus we will be left only with "x" on that side:

[tex]15-14On the left side, 15-14 is equal to 1:[tex]1This can be read as follows: "x is greater than 1".

And we can also write this solution with the x at the beginning:

[tex]x>1[/tex]

Answer: x>1

The plot below represents the function f ( x ) : 1 2 3 4 5 -1 -2 -3 -4 -5 1 2 3 4 5 -1 -2 -3 -4 -5 Evaluate f ( 3 ) : f ( 3 ) =

Answers

Solution

The function represented by the graph is

The root of the equation are -0.5 , 1.5

[tex]\begin{gathered} x=-0.5,x=2.5 \\ (x+0.5)(x-2.5) \\ x^2-2.5x+0.5x-1.25 \\ x^2-2x-1.25 \end{gathered}[/tex]

Therefore the function of x =

[tex]\begin{gathered} f(x)=x^2-2x-1.25 \\ f(3)=3^2-2(3)-1.25 \\ f(3)=9-6-1.25 \\ f(3)=1.75 \end{gathered}[/tex]

Hence the correct value of f(3) = 1.75

At KEY Middle School, there are 240 girls and 160 boys. What percent of all the students are girls?

Answers

Given;

Number of boys = 160

Number of girls = 240

Total number of students = 160 + 240 = 400

To find the percentage of all students that are girls, use the formula below:

[tex]\begin{gathered} \text{ \% of girls = }\frac{Number\text{ of girls}}{Total\text{ number}}\times\frac{100}{1} \\ \\ \text{ } \end{gathered}[/tex]

Therefore, we have:

[tex]\begin{gathered} \text{ \% of girls = }\frac{240}{400}\times\frac{100}{1} \\ \\ \text{ = }0.6\text{ }\times\text{ 100 = 60\%} \end{gathered}[/tex]

The percent of all the students that are girls is 60 percent.

ANSWER:

60%

girls + boys = 400
240/400 x 100/1
0.6 x 100 is 60
60%

What is the length, in whole meters, of the plastic edging that Amy needs to complete this project?

Answers

The curved path edges are in a form of two pairs of quadrants. One pair is shown below:

We can find the lengths of the inner and outer arcs using the radii of 4m and 6m respectively.

The formula to find the length of the arc of a quadrant is

[tex]L=\frac{1}{2}\pi r[/tex]

Length of inner arc:

[tex]\begin{gathered} l=\frac{1}{2}\times\pi\times4 \\ l=6.3m \end{gathered}[/tex]

Length of outer arc:

[tex]\begin{gathered} L=\frac{1}{2}\times\pi\times6 \\ L=9.4m \end{gathered}[/tex]

Hence, the length of the pair of arcs is

[tex]6.3+9.4=15.7m[/tex]

For the two pairs, we have the length to be

[tex]\begin{gathered} 15.7\times2 \\ =31.4m \end{gathered}[/tex]

Hence, it will take approximately 32 meters of plastic edging to complete the project.

Solve the following equation on the interval [0°, 360º). Round answers to the nearest tenth. If there is no solution, indicate "No Solution."2sec^2(x) - 13tan(x) = -13

Answers

Given

[tex]2\sec ^2(x)-13\tan (x)=-13[/tex]

Add 13 to both sides

[tex]\begin{gathered} 2\sec ^2(x)-13\tan (x)+13=-13+13 \\ 2\sec ^2(x)-13\tan (x)+13=0 \end{gathered}[/tex]

We have that

[tex]\sec ^2(x)=1+\tan ^2(x)[/tex]

So, substitute in the above equation

[tex]2(1+\tan ^2(x))-13\tan (x)+13=0[/tex]

Simplify

[tex]\begin{gathered} 2+2\tan ^2(x)-13\tan (x)+13=0 \\ 15+2\tan ^2(x)-13\tan (x)=0 \end{gathered}[/tex]

Reordering the equation

[tex]2\tan ^2(x)-13\tan (x)+15=0[/tex]

We get a quadratic equation, then solve by factoring

[tex](2\tan (x)-3)(\tan (x)-5)=0[/tex]

Separate the solutions

[tex]\begin{gathered} 2\tan (x)-3=0 \\ 2\tan (x)-3+3=0+3 \\ 2\tan (x)=3 \\ \frac{2\tan (x)}{2}=\frac{3}{2} \\ \tan (x)=\frac{3}{2} \end{gathered}[/tex]

And

[tex]\begin{gathered} \tan (x)-5=0 \\ \tan (x)-5+5=0+5 \\ \tan (x)=5 \end{gathered}[/tex]

Next, solve for x for each solution

[tex]\begin{gathered} \tan (x)=\frac{3}{2} \\ x=\tan ^{-1}(\frac{3}{2}) \\ x=56.3 \end{gathered}[/tex]

And

[tex]\begin{gathered} \tan (x)=5 \\ x=\tan ^{-1}(5) \\ x=78.7 \end{gathered}[/tex]

Answer:

x = 56.3° and x = 78.7°

Suppose that only two factories make Playstation machines. Factory 1 produces 70% of the machines and Factory 2 produces the remaining 30%. Of the machines produced in Factory 1, 2% are defective. Of the machines produced in Factory 2, 5% are defective. What proportion of Playstation machines produced by these two factories are defective? Suppose that you purchase a playstation machine and it is defective. What is the probability that it was produced by Factory 1?

Answers

Given:

Factory 1 produces 70%

Factor 2 produces 30%

Defective machines in factory 1 = 2%

Defective machines in factory 2 = 5%

Find-:

What is the probability that it was produced by Factory 1?

Explanation-:

Probability of machines produced by factory1

[tex]\begin{gathered} P(F_1)=70\% \\ \\ P(F_1)=\frac{70}{100} \\ \\ P(F_1)=\frac{7}{10} \\ \end{gathered}[/tex]

Probability of machines produced by factory 2

[tex]\begin{gathered} P(F_2)=30\% \\ \\ P(F_2)=\frac{30}{100} \\ \\ P(F_2)=\frac{3}{10} \end{gathered}[/tex]

Probability of factory 1 produced defective item,

[tex]\begin{gathered} P(\frac{x}{F_1})=2\% \\ \\ P(\frac{x}{F_1})=\frac{2}{100} \\ \\ P(\frac{x}{F_1})=\frac{1}{50} \end{gathered}[/tex]

Probability of factory 2 produced defective item,

[tex]\begin{gathered} P(\frac{x}{F_2})=5\% \\ \\ P(\frac{x}{F_2})=\frac{5}{100} \\ \\ P(\frac{x}{F_2})=\frac{1}{20} \end{gathered}[/tex]

So, the probability that randomly selected items was form factor 1.

[tex]P(\frac{F_1}{x})\text{ is}[/tex]

Now, apply Bayes theorem is:

[tex]P(\frac{F_1}{x})=\frac{P(F_1)P(\frac{x}{F_1})}{P(F_1)P(\frac{x}{F_1})+P(F_2)P(\frac{x}{F_2})}[/tex]

So, the value is:

[tex]\begin{gathered} =\frac{\frac{7}{10}\times\frac{1}{50}}{\frac{7}{10}\times\frac{1}{50}+\frac{3}{10}\times\frac{1}{20}} \\ \\ =\frac{\frac{7}{500}}{\frac{7}{500}+\frac{3}{200}} \\ \\ =\frac{\frac{7}{5}}{\frac{7}{5}+\frac{3}{2}} \\ \\ =\frac{\frac{7}{5}}{\frac{14}{10}+\frac{15}{10}} \\ \\ =\frac{\frac{7}{5}}{\frac{14+15}{10}} \\ \\ =\frac{7}{5}\times\frac{10}{29} \\ \\ =\frac{14}{29} \end{gathered}[/tex]

So, the probability is 14/29.



7) The point spreads on 12 football games for a season are:1, 3, 14,9,7,3,6, 27, 3, 13, 8, 17.a (3pts) Make a histogram for the data.1-511-1516-2021-2526-30Symmetric6-10b. (2 pts) Describe the distribution of the data, (Circle One)FrequencySkewed RightSkewed Left(2 pts)Which measure of center would be most accurate? (circle one)MeanMedianModeC.d. (2 pts) Which measure of spread would be most accurate? (circle one)RangeInterquartile rangeStandard Deviation

Answers

Table of frequencies.

Interval Frequency

1-5 4

6-10 4

11-15 2

16-20 1

21-25 0

26 - 30 1

Therefore the graph would be

B. As we can see from the graph it is skewed left.

C. Since the graph is skewed, the better option would be the median.

D. Since the graph is skewed, the better option would be the interquartile range.

Is an irrational number?

Answers

In this case a rational number is a number that could be represented as p/q where q is different to 0 in this case pi can't be represete

Suppose an auto racer won a 400 mile race with a time of 1:48:51. At one point the racer was 50 miles closer to the finish than the start. How far had the racer gone at that point?How far from the start was the racer? __ miles

Answers

To solve this problem, let's use the variables x and y to represent the distance of the racer to the start and to the finish, respectively.

If the total distance of the race is 400 miles, we have that the distance traveled by the racer until now (x) plus the distance he needs to travel to finish the race (y) is 400:

[tex]x+y=400[/tex]

Also, at one point the racer was 50 miles closer to the finish than the start, so at that point we have that he is farther away from the beginning (that is, x is 50 units bigger than y):

[tex]x=y+50[/tex]

Now, using this value of x in the first equation, we have:

[tex]\begin{gathered} (y+50)+y=400 \\ 2y+50=400 \\ 2y=350 \\ y=\frac{350}{2}=175 \end{gathered}[/tex]

Now, finding the value of x, we have:

[tex]x=y+50\to x=175+50\to x=225[/tex]

So the racer was 225 miles far from the start.

A doctor conducts an experiment to test new treatments for a medical condition. Out of the 6 volunteers in the experiment, 4 do not receive any treatment. What percent of the volunteers do not receive any treatment?

Answers

ANSWER

66.67%

EXPLANATION

We have that there were 6 volunteers in the experiment and 4 do not receive any treatment.

To find the percent of volunteers that do not receive any treatment, we have to divide the number of people that do not receive treatment by the total number of people that were in the experiment and multiply by 100.

That is:

[tex]\frac{4}{6}\cdot\text{ 100 = 66.67\%}[/tex]

That is the percent of volunteers that do not receive any treatment.

Select the quadrant or axis where each ordered pair is located on a coordinate plane.(9.5, 0)(-4, 7)(-1, -8)options:Quadrant IQuadrant IIQuadrant IIIQuadrant IV

Answers

The points (9.5, 0), (-4, 7), and (-1, -8) are plotted in the coordinate plane below:

Therefore, the quadrants where each point is located are:

• Quadrant I: (9.5, 0)

,

• Quadrant III: (-1, -8)

,

• Quadrant IV: (-4, 7)

A tank has a capacity of 13 gallons. When it is full, it contains 20% alcohol. How many gallons must be replaced with an 70% alcohol solution to give 13 gallons of 30% solution? Round your final answer to 1 decimal place if necessary.

Answers

Given:

A tank has a capacity of 13 gallons. When it is full.

The tank contains 20% alcohol.

We will find the number of gallons that must be replaced with a 70% alcohol solution to give 13 gallons of 30% solution

Let the number of gallons that must be replaced = x

so, there are x gallons with a 70% alcohol and (13 -x) with a 20% alcohol.

So, we can write the following equation:

[tex]70x+20(13-x)=30*13[/tex]

Solve the equation to find (x):

[tex]\begin{gathered} 70x+20*13-20x=30*13 \\ 50x+260=390 \\ 50x=390-260 \\ 50x=130 \\ x=\frac{130}{50}=2.6\text{ gallons} \end{gathered}[/tex]

So, the answer will be 2.6 gallons

In the given diagram, line segment BDbisects angle ABC. Segment BDis extended to E, where line segment ECis parallel to line segment AB.Write a two-column proof to show that AB/AD=BC/DC

Answers

Explanation:

From the question , we will utilize the concept of isosceles triangles

Concept:

The isosceles triangle theorem states that the angles opposite to the equal sides of an isosceles triangle are equal in measurement. So, in an isosceles triangle △ABC where AB = AC, we have ∠B = ∠C.

From the steps, we can see that

[tex]\angle2\cong\angle5(substituting\text{ property of congruency\rparen}[/tex]

Hence,

We can cconclude that the final answer is

[tex]EC=BC(properties\text{ of isosceles triangles\rparen}[/tex]

OPTION A is the correct answer

Answer:

EC = BC;  property of isosceles triangle

Hope this helps!

Step-by-step explanation:

12What mistake did the student make when solvingtheir two-step equation?(a)b) If correctly solved what should the value of be?

Answers

Given the equation:

[tex]\frac{x}{6}+3=-18[/tex]

(a) You can identify that the student applied the Subtraction Property of Equality by subtraction 3 from both sides of the equation:

[tex]\frac{x}{6}+3-(3)=-18-(3)[/tex]

However, the student made a mistake when adding the numbers on the right side.

Since you have two numbers with the same sign on the right side of the equation, you must add them, not subtract them and use the same sign in the result. Then, the steps to add them are:

- Add their Absolute values (their values without the negative sign).

- Write the sum with the negative sign.

Then:

[tex]\frac{x}{6}=-21[/tex]

(b) The correct procedure is:

1. Apply the Subtraction Property of Equality by subtracting 3 from both sides (as you did in the previous part):

[tex]\begin{gathered} \frac{x}{6}+3-(3)=-18-(3) \\ \\ \frac{x}{6}=-21 \end{gathered}[/tex]

2. Apply the Multiplication Property of Equality by multiplying both sides of the equation by 6:

[tex]\begin{gathered} (6)(\frac{x}{6})=(-21)(6) \\ \\ x=-126 \end{gathered}[/tex]

Hence, the answers are:

(a) The student made a mistake by adding the numbers -18 and -3:

[tex]-18-3=-15\text{ (False)}[/tex]

(b) The value of "x" should be:

[tex]x=-126[/tex]

School is making digital backups of old reels of film in its library archives the table shown approximate run Times of the films for a given diameter of film in the reel. Which of the following equations is a good model for the run time, y, as a function of the diameter, X?

Answers

One technique that you can apply when solving such a problem is trial and error. We try to use each equation to prove that a given value of x on the table given will correspond to the value of y on the table.

a) Let's try to put x = 3 for the first equation and we must get an answer equal to 2.25.

[tex]y=7.72(3)-29.02=-5.86_{}[/tex]

Since the value of y is not equal to 2.25 and the deviation is too large. this equation is not a good model,

b) We put x = 3 on the second equation and solve for y

[tex]y=-7.52(3)^2+0.19(3)+3.26=-63.85[/tex]

Since the value of y is not equal to 2.25 and the deviation is too large. this equation is not a good model,

c) We put x = 3 on the third equation and solve for y,

[tex]y=0.4(3)^2+0.79(3)-4.93=1.04[/tex]

Again, the value that we get is not equal to 2.25, hence, this equation is not a good model. But since its value is close to 2.25, we try to other values of x. If x = 5, we get

[tex]y=0.4(5)^2+0.79(5)-4.93=9.02[/tex]

which has a slight deviation on the given value of y on the table for x = 5. let's try for x = 7. We have

[tex]y=0.4(7)^2+0.79(7)-4.93=20.2[/tex]

and the answer has a small deviation compared to the actual value given. The other values of x can again be put on the equation and check their corresponding value of y, and the resulting values are as follows

[tex]\begin{gathered} y=0.4(8)^2+0.79(8)-4.93=26.99 \\ y=0.4(12)^2+0.79(12)-4.93=62.15 \\ y=0.4(14)^2+0.79(14)-4.93=84.53 \end{gathered}[/tex]

And as you can see, the deviation of values from the table to calculated becomes smaller. Hence, this is the best model.

d) We put x = 3 on the third equation and solve for y,

[tex]y=4.19(1.02)^3=4.45_{}_{}[/tex]

Again, the value that we get is not equal to 2.25, hence, this equation is not a good model. But since its value is close to 2.25, we try to other values of x. If x = 5, we get

[tex]y=4.19(1.02)^5=4.63[/tex]

where the answer's deviation is too large compared to the value of y if x = 5 on the table given.

Based on the calculations used above, the best equation that can be a good model is equation 3.

if FE measures 20 centimeters, the approximate area of circle B is what

Answers

If FE measures 20 cm, then the area is 314 cm², if BE measure 3.5 cm, then the area is 38.5 cm², if AB measures 11 cm, then the area is 380 cm² and is EF measures 12 cm, then the area is 113 cm².

Area of a circle:

A = π r²

r = 1 /2 of diameter.

FE is the diameter

r = 20 / 2

r = 10 cm

Area of circle using FE:

A =  π ( 10 )² = π × 100 = 314 cm²

BE is a radius:

Area = π × 3.5² = π × 12.25 = 38.465

A = 38.5 cm²

AB is a radius:

Area = π × 11²
A = π × 121

A = 379.94 = 380 cm²

EF is a diameter:

r = 12 / 2 = 6 cm

Area = π × 6²

A = π × 36

A = 113.04 = 113 cm²

Therefore, if FE measures 20 cm, then the area is 314 cm², if BE measure 3.5 cm, then the area is 38.5 cm², if AB measures 11 cm, then the area is 380 cm² and is EF measures 12 cm, then the area is 113 cm².

Learn more about area here:

https://brainly.com/question/25292087

#SPJ9

Your question was incomplete, Please refer the content below:

1) if FE measures 20 centimeters, the approximate area of circle B is what?

2) if BE measures 3.5 centimeters, the approximate area of circle B is what?

3) if AB measures eleven centimeters, the approximate area of circle B is what

4) If EF measures twelve centimeters, the approximate area of circle B is what?

2. The water level in a reservoir is now 52 meters. Which equation can be used to find the initial depth, d, if this is the water level after a 23% increase? * O 0.23. d = 52 O d = 52 · 0.23 O 1.23. d = 52 O d = 52. 1.23

Answers

Answer:

1.23d = 52

Explanation:

If 52 meters is the water level after a 23% increase, then we can say that the initial depth d added to the 23% of d is equal to 52 meters. So:

d + 23%d = 52 meters

Since 23% is equivalent to 0.23, we get:

d + 0.23d = 52

Finally, adding the like terms, we get:

(1 + 0.23)d = 52

1.23d = 52

So, the equation is:

1.23d = 52

Write down 2 fractions where the denominator of one is a multiple of the denominator of other

Answers

The two fractions are 1/3 and 1/6.

What is a fraction?

A fraction has two parts: Numerator and Denominator.

It is in the form of a Numerator / Denominator. A fraction is a numerator divided by the denominator.

We need to write 2 fractions where the denominator of one is a multiple of the denominator of the other.

Let's consider the one fraction as;

1/3

Then another one must be multiple of the denominator of the other.

So, 1/6

We see that "the denominator of one is a multiple of the denominator of other".

Thus the two fractions are 1/3 and 1/6.

Learn more about fractions here:

brainly.com/question/10354322

#SPJ1

Margo borrows $1400, agreeing to pay it back with 6% annual interest after 16 months. How much interest will she pay?

Answers

Given,

The principal amount is $1400.

The rate of interest is 6%.

The time period is 16 months.

Required

The interest paid by Morrow.

The simple interest is calculated as,

[tex]Simple\text{ interest=}\frac{P\times R\times T}{100}[/tex]

Substituting the values then,

[tex]\begin{gathered} S.I=\frac{1400\times6\times16}{100\times12} \\ S.I=14\times2\times4 \\ S.I=56\times2 \\ S.I=112 \end{gathered}[/tex]

Hence, the interest she will pay is $112.

The terminal side contains the point (-6, -8). Find tan θ.Question 18 options:.751.3-.75-1.3

Answers

Given:

The terminal side contains the point (-6, -8).

To find:

[tex]\tan \theta[/tex]

Here, x= -6 and y= -8.

Using the formula,

[tex]\begin{gathered} \tan \theta=\frac{y}{x} \\ \tan \theta=\frac{-8}{-6} \\ \tan \theta=1.3 \end{gathered}[/tex]

Hence, the answer is,

[tex]1.3[/tex]

Jody invested $4400 less in account paying 4% simple interest than she did in an account paying 3 percent simple interest. At the end of the first year, the total interest from both accounts was $592. find the amount invested in each account

Answers

The rule of the simple interest is

[tex]I=P\times R\times T[/tex]

P is the initial amount

R is the rate in decimal

T is the time

Assume that she invested $x in the account that paid 3% simple interest

then she invested x - 4400 dollars in the account that paid 4% simple interest

Then let us find each interest, then add them, equate the sum by 592

[tex]\begin{gathered} P1=x-4400 \\ R1=\frac{4}{100}=0.04 \\ T1=1 \\ I1=(x-4400)\times0.04\times1 \end{gathered}[/tex]

Let us simplify it

[tex]\begin{gathered} I1=0.04(x)-0.04(4400) \\ I1=0.04x-176 \end{gathered}[/tex][tex]\begin{gathered} P2=x \\ R2=\frac{3}{100}=0.03 \\ T2=1 \\ I2=x\times0.03\times1 \\ I2=0.03x \end{gathered}[/tex]

Since the total interest is $592, then

[tex]\begin{gathered} I1+I2=592 \\ 0.04x-176+0.03x=592 \end{gathered}[/tex]

Add the like terms on the left side

[tex]\begin{gathered} (0.04x+0.03x)-176=592 \\ 0.07x-176=592 \end{gathered}[/tex]

Add 176 to both sides

[tex]\begin{gathered} 0.07x-176+176=592+176 \\ 0.07x=768 \end{gathered}[/tex]

Divide both sides by 0.07 to find x

[tex]\begin{gathered} \frac{0.07x}{0.07}=\frac{768}{0.07} \\ x=10971.42857 \end{gathered}[/tex]

Then She invested about 10971 dollars in the account of 3%

Since 10971 - 4400 = 6571

Then she invested about

graph the system of linear inequalities.x + 2y ≥ 2-x + y ≤ 0

Answers

INFORMATION:

We have the next system of equations

[tex]\begin{gathered} x+2y\ge2 \\ -x+y\leq0 \end{gathered}[/tex]

And we must graph it

STEP BY STEP EXPLANATION:

To graph the system, we need to graph first the two inequalities as equations. So, we would have

[tex]\begin{gathered} x+2y=2 \\ -x+y=0 \end{gathered}[/tex]

- x + 2y = 2:

To graph it, we can find the x and y intercepts.

x intercept:

To find it, we need to replace y = 0, and solve for x

[tex]\begin{gathered} x+2(0)=2 \\ x=2 \end{gathered}[/tex]

y intercept:

To find it, we need to replace x = 0, and solve for y

[tex]\begin{gathered} 0+2y=2 \\ y=1 \end{gathered}[/tex]

So, the graph would be a line that passes through the points (2, 0) and (0, 1).

Since the symbol of this inequality is ≥, the graph would be the values that are on the line and above it.

- -x + y = 0:

To graph it, we can rewrite the equation as

[tex]y=x[/tex]

And this is the identity line.

So, since the symbol of this inequality is ≤, the graph would be the identity line and the values below it.

Finally, the graph of the system would be the common part of the graph of each inequality

So, the graph of the system is the part colored in red and blue at the same time

ANSWER:

Vector u has initial point at (8, 6) and terminal point at (–6, 12). Which are the magnitude and direction of u?

Answers

SOLUTION

Write out the given point

[tex](8,6)\text{ and (-6,12)}[/tex]

The magnitude of the vertor u is the distance between the two point.

[tex]\begin{gathered} \text{distance = }\sqrt[]{(x_2-x_1)^2+(y_2-y_1)^2} \\ \text{Where } \\ x_2=-6,x_1=8 \\ y_2=12,y_1=6 \end{gathered}[/tex]

Substitute into the formula, we have

[tex]\begin{gathered} \mleft\Vert u\mleft\Vert=\sqrt[]{(-6-8)^2+(12-6)^2}\mright?\mright? \\ \mleft\Vert u\mleft\Vert=\sqrt[]{(-14)^2+6^2}\mright?\mright? \\ \mleft\Vert u\mleft\Vert=\sqrt[]{169+36}=\sqrt[]{232}\mright?\mright? \end{gathered}[/tex]

Hence

|| u || =15.23

The magnitude of the vector is 15.232

Then the direction is obtain by using the formula

[tex]\begin{gathered} \tan \theta=\frac{y_2-y_1}{x_2-x_1} \\ \text{Then } \\ \tan \theta=\frac{12-6}{-6-8}=\frac{6}{-14}=-0.4286 \end{gathered}[/tex]

Then we have

[tex]\tan \theta=-0.4286[/tex]

take inverse tan of the equation above, we have

[tex]\begin{gathered} \theta=\tan ^{-1}(-0.4286) \\ \theta=156.801^0 \end{gathered}[/tex]

Hence

The direction is of u is 156.801°

Answer: Second Option

Other Questions
Can I find a tutor to help me with question? Calculate the number of molecules in 3.00 L of CO gas at STP. What is an equation of the line that passes through the point (4,2)(4,2) and is perpendicular to the line 4x+3y=214x+3y=21? Janet is working on a presentation that focuses on a specific period in the history of photography when documentary and photojournalistic photography were in vogue, with important photographic essays appearing in Life magazine and elsewhere and artists and mass media outlets beginning to use color on a regular basis. What historical period of photography is Janet doing her presentation on?Modern Photography 1900-1945Digital Photography 2000-presentEarly Photography 1839-1900Contemporary Photography 1945-2000 Translate solve and check:"Twice a number, increased by three times the difference of the number and 3 is 11" A linear function contains the following points.XyWhat are the slope and y-intercept of this function?A. The slope is 4.The y-intercept is (0, -1).5B. The slope is.The y-intercept is (0, -1).C. The slope is.The y-intercept is (-1,0).D. The slope is.0-1The y-intercept is (0, -1).53 Preform the indicated Operation g(n)=2n^2-4nh(n)=n-1find g(h(1-b)) Translation of a PointWhat is the image point of (5,-5) after a translation right 2 units and up 3 units?Submit Answer Roberto creates an animation titled Flying through an Atom. The animation makes it seem as if the viewer passes all the way through an atom, starting from one side, then through the atoms center, and then leaving through the other side.What is the best description of the order of events in the animation? a+ 1 + 2a + 2a, a3 - 1 and a+a2 +1 determine the type and key parts of the graph of the second equation what happens when mix vinegar and baking soda???is for science project How to calculate electric field on a charge How much material will Dmitris nom need for the tent, including the floor? what do new cells do in a chicken egg Hi can you help me find the correct answer to this? Complete the equation of the line through (-10,-7) and (-5, -9).Use exact numbers.y = need to show 1,242 23 = and 732 x 268 = show answers on graph Graph each line given the slope and y-intercept.Label each one One spring day, Evan noted the time of day and the temperature, in degrees Fahrenheit. His findings are as follows: At 6 a.m., the temperature was 58 F. For the next 3 hours, the temperature rose 1 per hour. For the next 4 hours, it rose 2 per hour. The temperature then stayed steady until 6 p.m. For the next 3 hours, the temperature dropped 2 per hour. The temperature then dropped steadily until the temperature was 62 at midnight. On the set of axes below, graph Evan's data.